Twisted Restraints

Geometry Level 3

The above shows a rectangle A B C D ABCD with an area of 672, a diagonal C B CB of length 42.
Given that A E C B AE \perp CB and D F C B DF \perp CB , find C E CE .

Give your answer to 1 decimal place.

You may use a calculator for the final step of your calculation.


The answer is 7.4.

This section requires Javascript.
You are seeing this because something didn't load right. We suggest you, (a) try refreshing the page, (b) enabling javascript if it is disabled on your browser and, finally, (c) loading the non-javascript version of this page . We're sorry about the hassle.

3 solutions

Relevant wiki: Length and Area Problem Solving

Area A B C D = 672 \text{Area} \ ABCD = 672

Area A C B = 672 2 = 336 \text{Area} \ ACB = \frac{672}{2} = 336

A E = 336 × 2 42 = 16 AE = \frac{336 \times 2 }{42} = 16

Now we can add one more diagonal

A G = G D = 21 AG = GD = 21

Step into Pythagorean theorem

A E 2 + E G 2 = A G 2 AE^2 + EG^2 = AG^2

1 6 2 + E G 2 = 2 1 2 16^2 + EG^2 = 21^2

E G = G F = 185 EG = GF = \sqrt{185}

Looking at the figure above, we can conclude that

C E = 42 2 185 2 CE = \frac{42 - 2\sqrt{185}}{2}

C E = 2 ( 21 185 ) 2 CE = \frac{2(21-\sqrt{185})}{2}

C E = 21 185 CE = 21 - \sqrt{185}

C E 7.4 CE \approx 7.4

the rectangle is ABDC not ABCD :)

Joelle Farah - 4 years, 10 months ago

Log in to reply

XD it was a little mistake

Jason Chrysoprase - 4 years, 10 months ago

Log in to reply

@Jason Chrysoprase hahaha it's okay :)

Joelle Farah - 4 years, 10 months ago

Yes it's true, i failed this problem because i running out of time :(

Jason Chrysoprase - 4 years, 10 months ago

I liked your approach. Voted up.

Niranjan Khanderia - 4 years, 9 months ago

Here's a less clever method based on similar triangles. First, note that we can express A A C B A_{\triangle ACB} in two ways: Thanks to diagonal symmetry, A A C B = 672 2 = 336 A_{\triangle ACB} = \frac{672}{2} = 336 . But since A E C B AE\perp CB , we also know that A A C B = 1 2 42 A E = 336 A_{\triangle ACB} = \frac{1}{2}\cdot 42\cdot AE = 336 . Thus, A E = 16 AE = 16 .

Now, since C B D A C B \angle CBD \cong \angle ACB , we know C A E C B A \angle CAE \cong \angle CBA . Thus, A C E B A E \triangle ACE\sim\triangle BAE , and it follows that C E A E = A E C B C E \frac{CE}{AE} = \frac{AE}{CB - CE} C E 16 = 16 42 C E \frac{CE}{16} = \frac{16}{42 - CE} C E 2 42 C E + 256 = 0. CE^2 - 42CE + 256 = 0. From the quadratic formula, we find C E 7.4 CE \approx 7.4 or C E 34.6 CE \approx 34.6 . But since 42 = C E + E F + F B = 2 C E + E F 42 = CE + EF + FB = 2CE + EF , in order for E F EF to be non-negative, we must reject the larger solution and conclude C E 7.4. CE \approx 7.4.

Actually this was my first solution :)

I still beleive that there is an easy way, so i stare 2 hours at the problem, then i realize that you can use easier method ( My posted solution )

It's really great to see someone post something that i was planned :)

Jason Chrysoprase - 4 years, 10 months ago

It sounds strange but shouldn't the area be expressed as S S ? :D

A Former Brilliant Member - 4 years, 10 months ago

L e t A C = x < A B = y . x 2 + y 2 = 4 2 2 , x y = 672. A l s o B C A E = 672 , A E = 672 42 = 16. ( x + y ) 2 = 3108 ( x y ) 2 = 420. x = 777 105 , C E 2 = A C 2 A E 2 = x 2 1 6 2 = 54.738. S o C E = 7.398 Let\ AC=x \ <\ AB=y.\\ x^{2}+y^{2}=42^2,\ \ \ \ \ \ xy=672.\ \ \ \\ Also\ BC*AE=672,\ \ \ \ \ \implies\ AE=\dfrac{672}{42}=16.\\ \therefore\ (x+y)^2=3108\ \ \ \ \ \ \ (x-y)^2=420.\\ \implies\ x=\sqrt{777}-\sqrt{105},\\ CE^2=AC^2- AE^2=x^2-16^2=54.738.\\ So\ CE=7.398

0 pending reports

×

Problem Loading...

Note Loading...

Set Loading...